Перейти к содержанию

Мощность лампы и расчет Ra от Ri


Рекомендуемые сообщения

... Внутреннее сопротивление лампы: Ri - это величина по ПЕРЕМЕННОМУ ТОКУ ! ...

 

Так я не спорю. Вот она мне и не нужна эта величина. Статические параметры ни к чему. При разном значении рабочей точки, всегда получаются разные значения сопротивлений. Их проще высчитать из постоянных величин до большой точности, без различных подгонок в виде 2 - 4 * Ri, а для пентодов и тетродов 0,1 - 0,2 * Ri.

Ссылка на комментарий
Поделиться на другие сайты

  • Ответов 188
  • Создана
  • Последний ответ

Топ авторов темы

... Вот страничка из "Радиотехники для радиолюбителей ...

 

А что вам дает это значение Ri? По нему ведь не возможно точно высчитать анодное сопротивление.

Ссылка на комментарий
Поделиться на другие сайты

Ничего смешного нет. Попробуйте сами посчитать из постоянных величин. Думаю быстро поймете о чем я толкую.

  • Ты не прав! 1
Ссылка на комментарий
Поделиться на другие сайты

Давайте начнем с малого.

В общем, не буду тянуть время. Пример расчета резистивного каскада, основанного на постоянных величинах, без Ri.

О точности расчета можете судить сами, ознакомившись с материалом.

 

На прикрепленной картинке в рис.1 указаны данные с ВАХ одной из ламп. Без разницы, какую вы сами выберете для ваших нужд. Исключение: ВАХ должны быть не самопальные, а паспортные.

Картинка составлена по общепринятому стандарту.

 

Итак. Не буду приводить полный проводимый мной расчет, так как он большой.

Мне необходимо усиление равное Ku=20. Выбираю подходящую лампу. На графике, на вскидку, выбираю оптимальную рабочую точку Op так, что бы усиление в Ku=20 приходилось на точку А.

Где определяем: значение Io, величину тока Im, напряжение рабочей точки Uo, и напряжение Um.

Больше чертежные работы мне не нужны. Дело за математикой.

Определяем значение сопротивления по постоянному току лампы в точке Op.

У него есть обозначение - Ro.

Ro = Uo/Io, (190/0,004 = 47500)

 

Очевидно, что этим значением никто не пользуется. Вот оно как раз необходимо для расчета постоянных параметров лампы. Мы же создаем условия своей жизни постоянной работой, постоянным жильем..., что бы наше "переменное" состояние находилось в рамках постоянных величин. Иначе, в погоне за переменными составляющими, можно лишиться постоянных условий. Так и здесь.

 

Далее. Ra вычисляется самым обычным способом.

Ra = dUa/dIa, (20/0,001 = 20000)

Что бы проверить правильность вычисления Ra и узнать напряжение питания резистивного каскада, прибегнем к формуле:

Ea = Uo+(Ra x Io), 190+(20000 x 0,004) = 270

Uo = Ea-(Io x Ra), 270-(0,004 x 20000) =190

 

Но ведь здесь нет значения Ro.

Смотрим формулы:

Ea = (Ro+Ra)x Io, (47500+20000)x 0,004 = 270

Ea = (Ro x Io)+(Ra x Io) =270

Uo = Ea x (Ro /(Ro+ Ra)), 270 x (47500/(47500 + 20000)) =190

 

И в дополнении. При расчете выходной лампы, допустим имеем питание в 300 Вольт.

Рис.2. И значение рабочей точки тоже находится на 300 В.

Вычисление Ra производится также: Ra = dUa/dIa.

Тогда каким образом отрицательная полуволна сигнала превысит 300 вольт, точка В? Не ужели не видно разницы на графиках?

И как сказал "nevod" - " 1. В радиоэлектронике открытия случаются с завидной регулярностью. Как раз потому что теория за уши притянута."

 

Пробуйте, считайте.

post-31352-0-60730500-1472821561_thumb.jpg

Ссылка на комментарий
Поделиться на другие сайты

Анатолий, Вы упускакте самый важный момент!

В большинстве случаев важен не столько коэффициент усиления, сколько достижение определённой выходной мощности на конкретной (часто - индуктивной) нагрузке. От этих двух параметров зависит выбор режима по постоянному току и необходимый коэффициент усиления.

 

... И в дополнении. При расчете выходной лампы, допустим имеем питание в 300 Вольт. Рис.2. И значение рабочей точки тоже находится на 300 В. ...

 

В резистивном каскаде такого быть не может, ведь на анодном резисторе должно падать напряжение не менее амплитуды выходного сигнала.

Такое может быть при индуктивной нагрузке с нулевым омическим сопротивлением и отсутствием оттока магнитного поля вовне. Я с трудом могу представить себе подобное в реале.

Ссылка на комментарий
Поделиться на другие сайты

... сколько достижение определённой выходной мощности ...

 

Здесь речь о резистивном каскаде, по рис.1. Что же касается:

... В резистивном каскаде такого быть не может, ведь на анодном резисторе должно падать напряжение не менее амплитуды выходного сигнала. ...
, то Ea - Uo, то получим как раз напряжение падения (270-190=80).

А по рис.2, о выходном каскаде на индуктивную нагрузку.

Какая разница какой каскад. Может и считают все наклон синусоиды от ее середины отрицательной частью, но, забросайте меня камнями, это не так.

Ссылка на комментарий
Поделиться на другие сайты

... на конкретной (часто - индуктивной) нагрузке. ...

Прошу прощения, конечно же на комплексной нагрузке.

Ссылка на комментарий
Поделиться на другие сайты

Вторая история. Расчет выходного трансформатора.

Этот расчет у меня уже более 15 лет. Из какой книги взял, уже и не вспомню. Но точно знаю, что это работает. Лет 7-10 назад делал 5 усилителей с расчетом по такой версии выходников.

Просто его немного подкорректировал, хотя способ довольно устаревший.

Итак, продолжим.

dU = Uo - Umin, (290-70=220). Uo - рабочая точка. Umin - минимальное напряжение на сгибе ВАХ при максимальном токе.

dI = Imax - Io, (0,115 - 0,045=0,07),

Ra = dU/dI = 220/0.07=3100,

P = Uo x Io = 290 x 0.045 = 13,

получим новое значение: dU = корень из (P x Ra) = 200, что ближе к оптимальному.

далее:

коэффициент трансформации: n = корень из (Rn/Ra) = корень из (4/3100) = 0.036,

количество витков на 1 вольт: w = dU x n, (200 x 0.036) = 7.2,

w1 = 200 x 7.2 = 1440, я округлил в большую сторону. Сделал запас, что бы можно было проверять и на других выходных лампах.

w4 = 7.2 x 7.2 = 51.84

Как мог.

Вот кстати, от этого расчета и началась эпопея расчетов.

Ссылка на комментарий
Поделиться на другие сайты

P = Uo x Io = 290 x 0.045 = 13 - это мощность рассеиваемого на аноде тепла

Ra у Вас это сопротивление анодного резистора. Или сопротивление первичной обмотки трансформатора постоянному току. А Rn - сопротивление вторичной по переменному току. Какая связь? Постоянный ток не трансформируется.

Анатолий, без обид, ситуация напоминает начало старого анекдота: Стоит гаишник на посту и пытается понять, почему 2х2 равно 2+2, а 3х3 не равно 3+3.

Вы случайно получили при таком расчёте в какой-то степени правдоподобные параметры трансформатора. Не сгорел сразу - уже хорошо. Но не факт, что оптимален именно такой. Потому Вас и просили показать замеры на частоте 20 Гц.

Ссылка на комментарий
Поделиться на другие сайты

... Ra у Вас это сопротивление анодного резистора. Или сопротивление первичной обмотки трансформатора постоянному току. А Rn - сопротивление вторичной по переменному току. ...

 

Здесь нет никакой ошибки.

Ra - реактивное сопротивление первичной, =3100. И здесь получилось активное сопротивление ra=100 Ом. Так как Ra - сопротивление рассчитанное, оно не является достаточно точным значением. Не возможно определить его абсолютное значение.

Rn - сопротивление катушки динамика. Так же имеет два значения. 1. По постоянному току, на пример =4 Ом. 2. И импеданс для 4-х омной катушки равный 3.5-3.6 Ом.

Потому формула: n = корень из (Rn/Ra) = корень из (4/3100) = 0.036, является, по сути, явно приближенной. Но достаточно более-менее пригодна для расчета. Пример: корень из (3,6/3100) = 0,034.

Мотать количество витков точь-в-точь не имеет смысла.

  • Ты не прав! 1
Ссылка на комментарий
Поделиться на другие сайты

... Для чего изобретать ВЕЛОСИПЕД? ...

 

Здесь речь не о изобретении велосипеда, а о его тех. характеристиках и улучшении его использования. Это как телефон в 20-х годах прошлого столетия и современный.

Ссылка на комментарий
Поделиться на другие сайты

Гость
Эта тема закрыта для публикации ответов.
  • Последние посетители   0 пользователей онлайн

    • Ни одного зарегистрированного пользователя не просматривает данную страницу

×
×
  • Создать...